2
$\begingroup$

What form does the Riemann hypothesis have for a global L-function?

  • 0
    [Here](http://math.stackexchange.com/questions/15350) is a thread you might want to see...2012-05-02
  • 0
    @J.M., that's very helpful. Thanks a lot.2012-05-02

1 Answers 1